Difference between revisions of "1997 PMWC Problems/Problem T10"

Line 1: Line 1:
[http://home.comcast.net/~wolfand/  1997 PMWC Problem T10]
+
==Problem==
 +
 
 +
The twelve integers 1, 2, 3,..., 12 are arranged in a circle such that the difference of any two adjacent numbers is either 2, 3 or 4. What is the maximum number of the difference '4' can occur in any such arrangement?
 +
 
 +
==Solution==
 +
 
 +
{{solution}}
 +
 
 +
==See Also==
 +
 
 +
{{PMWC box|year=1997|num-b=T9|after=Last<br />Problem}}

Revision as of 00:04, 13 May 2011

Problem

The twelve integers 1, 2, 3,..., 12 are arranged in a circle such that the difference of any two adjacent numbers is either 2, 3 or 4. What is the maximum number of the difference '4' can occur in any such arrangement?

Solution

This problem needs a solution. If you have a solution for it, please help us out by adding it.

See Also

1997 PMWC (Problems)
Preceded by
Problem T9
Followed by
Last
Problem
I: 1 2 3 4 5 6 7 8 9 10 11 12 13 14 15
T: 1 2 3 4 5 6 7 8 9 10